what does newton's universal law of gravitation explain? (select all the apply) explains why an apple falls at a constant rate explains the origin of mass explains the motion of the moon and other planets explains or implies kepler's laws

Answers

Answer 1

Newton's universal law of gravitation explains the motion of the moon and other planets, and implies Kepler's laws. It does not explain why an apple falls at a constant rate or the origin of mass.

Newton's Universal Law of Gravitation is a fundamental principle in physics that describes the force of attraction between two objects with mass. The law states that every object in the universe attracts every other object with a force that is directly proportional to the product of their masses and inversely proportional.

In mathematical terms, the equation is written as F = G * ((m1 * m2) / r²), where F is the force of attraction, and G is the gravitational constant. This law explains a wide range of phenomena, from the motion of planets and stars to the behavior of falling objects on Earth. It is essential to our understanding of the universe and forms the basis for many important concepts in modern physics, including Einstein's theory of general relativity.

To know more about Newton's universal law visit here:

brainly.com/question/3159150

#SPJ4


Related Questions

A submarine is a boat that can travel below the surface of the sea.
A submarine is 20m below the surface of the sea. The pressure due to the sea water at this
depth is P.
On another day, the submarine is 26 m below the surface of fresh water. The density of sea water is 1.3 times the density of fresh water.
What is the pressure due to the fresh water at a depth of 26 m?

Answers

The submarine is 26 m below the surface of fresh water. The density of sea water is 1.3 times the density of fresh water. The pressure due to fresh water at a depth of 26 m is 255660 Pa.

The pressure due to the sea water at a depth of 20 m can be calculated using the formula:

P = ρgh

where ρ is the density of the sea water, g is the acceleration due to gravity, and h is the depth of the submarine.

Assuming a density of sea water to be 1030 kg/m³ and g to be 9.81 m/s², we get:

P = 1030 x 9.81 x 20 = 202086 Pa

Now, to calculate the pressure due to fresh water at a depth of 26 m, we can use the same formula with the density of fresh water and the given depth:

P = ρgh

Assuming a density of fresh water to be 1000 kg/m³, and g to be 9.81 m/s², we get:

P = 1000 x 9.81 x 26 = 255660 Pa

For more such questions on pressure, click on:

https://brainly.com/question/29577233

#SPJ11

a particular balloon is designed by its manufacturer to be inflated to a volume of no more than l. if the balloon is filled with l helium at sea level, is released, and rises to an altitude at which the atmospheric pressure is only mm hg, will the balloon burst? (assume temperature is constant.)

Answers

The external pressure at the altitude is only 1474 mmHg.

To determine if the balloon will burst, need to compare the pressure inside the balloon to the external pressure at the altitude where it reaches, can use the ideal gas law to relate the pressure, volume, and temperature of a gas:

PV = nRT

where P is the pressure, V is the volume, n is the number of moles of gas, R is the ideal gas constant, and T is the absolute temperature.

Since the balloon is filled with helium, we can assume that the gas behaves ideally. If the balloon is filled to a volume of l at sea level, then  find the initial pressure of the helium using the ideal gas law and the conditions at sea level (assuming standard atmospheric conditions):

P₁V = nRT

P₁ = nRT / V = (1 mol * 8.31 J/(mol*K) * 293 K) / l

P₁ ≈ 2.42 atm = 1847 mmHg

Therefore, the initial pressure inside the balloon is approximately 2.42 atm or 1847 mmHg.

As the balloon rises to an altitude where the atmospheric pressure is only P2 mmHg, the pressure inside the balloon decreases due to the decreasing external pressure. If the external pressure drops below the pressure inside the balloon, the balloon will expand and eventually burst.

To find the pressure inside the balloon at the higher altitude, use the ideal gas law again and assume that the volume of the helium remains constant (since the balloon is designed not to exceed a volume of l):

P₂V = nRT

P₂ = nRT / V = (1 mol * 8.31 J/(mol*K) * 293 K) / l

P₂ ≈ 2.42 atm * (760 mmHg / 1 atm) * (P2 / 760 mmHg)

Solving for P2, we get:

P₂ ≈ 1474 mmHg

Since the external pressure at the altitude would be only 1474 mmHg, which is less than the pressure inside the balloon of 1847 mmHg, the balloon will not burst. However, the balloon will expand due to the decreasing external pressure, and the volume of helium inside the balloon will increase.

To know more about pressure

https://brainly.com/question/2036830

#SPJ4

a 4.0 kg particle is moving horizontally with a speed of 5.0 m/s when it strikes a vertical wall. the particle rebounds with a speed of 3.0 m/s. what is the magnitude of the impulse delivered to the particle?

Answers

The magnitude of the impulse delivered to the particle is 32 kg m/s when its mass is 4 kg, horizontal speed is 5.0 m/s and the rebound speed is 3.0 m/s.


First, let's find the initial momentum of the particle:
Initial momentum (p_initial) = mass × initial velocity
p_initial = 4.0 kg × 5.0 m/s
p_initial = 20 kg m/s


Next, let's find the final momentum of the particle after it rebounds. Since the particle moves in the opposite direction after rebounding, its final velocity will be negative:
Final momentum (p_final) = mass × final velocity
p_final = 4.0 kg × (-3.0 m/s)
p_final = -12 kg m/s


Now, we can find the change in momentum, which is the impulse delivered to the particle:
Impulse = p_final - p_initial
Impulse = (-12 kg m/s) - (20 kg m/s)
Impulse = -32 kg m/s


The magnitude of the impulse is the absolute value of the impulse, so:
Magnitude of impulse = |-32 kg m/s|
Magnitude of impulse = 32 kg m/s


Know more about impulse here:

https://brainly.com/question/30466819

#SPJ11

you decide to re-orient the wire to minimize the magnetic force acting on the wire. in what direction(s) could the current flow?

Answers

You may reduce the magnetic force acting on a wire by adjusting its orientation and/or the direction of the current flowing through it. One method for reducing magnetic force is to position.

the wire perpendicular to the magnetic field's direction. The magnetic force on the wire would be decreased in this design. You might even adjust the direction of the electricity flowing through the wire. The force on the wire will be greatest if the current flows in the same direction as the magnetic field. Yet, reversing the direction of the current reduces the stress on the wire. As a result, the current might run perpendicular to the magnetic field or parallel to it. the opposite direction to the original current flow to minimize the magnetic force acting on the wire.

learn more about   force here:

https://brainly.com/question/13191643

#SPJ4

a 1100 kg car travels at 25 m/s and then quickly stops in 4.2 s to avoid an obstacle. what is the initial momentum of the car?

Answers

The initial momentum of the car is 27500 kg m/s.

In the case of the car traveling at a velocity of 25 m/s, it has a certain momentum. When the car comes to a stop, its momentum changes to zero, since the velocity becomes zero. This change in momentum occurs due to a force that acts on the car to bring it to a stop.  The initial momentum of the car can be calculated using the formula:

p = m*v

where p is the momentum, m is the mass, and v is the velocity.

Given that the mass of the car is 1100 kg and its velocity is 25 m/s, we have:

p = 1100 kg * 25 m/s = 27500 kg m/s

To know more about momentum, here

brainly.com/question/30882550

#SPJ4

How does a balanced chemical equation show the law of conservation
of matter?

Answers

A balanced chemical equation demonstrates the Law of Conservation of Mass because it shows that the same amount of mass is present on both sides of the equation. This means that the total mass of the reactants is equal to the total mass of the products.

describe photoelectric effect. group of answer choices photoelectric effect is the hypothesis that many metals emit electrons when light of high enough energy shines upon them. photoelectric effect is the observation that many metals emit electrons when light of high enough energy shines upon them. photoelectric effect is the observation that many metals absorb electrons when light of high enough energy shines upon them

Answers

The correct answer is: "Photoelectric effect is the observation that many metals emit electrons when light of high enough energy shines upon them."

The photoelectric effect is a phenomenon in which electrons are emitted from the surface of a material when it is exposed to light of a sufficiently high frequency or energy.

When light is incident on the surface of a metal, the photons of the light interact with the electrons in the metal. If the energy of the photons is greater than the work function of the metal

Which is the minimum amount of energy required to remove an electron from the metal, then electrons are emitted from the metal surface. These emitted electrons are called photoelectrons.

To learn more about Photoelectric effect here:

https://brainly.com/question/26465043

#SPJ11

URGENT!!!
A small glass ball is rubbed with a piece of silk, giving the ball a charge of 1.0 x 10^-8 C. determine the magnitude of the force due to the Earth's magnetic field if the ball is thrown to the west with a velocity 8.0 m/s .
The earth's magnetic field is 5.0 x 10^-5 T

Answers

Answer:

[tex]F = 3\times 10^{-12} \ N [/tex]

Explanation:

To find:-

The magnitude of force due to Earth's magnetic field.

Answer:-

We are here given that, a ball having a charge of 1*10-C is thrown with a velocity of 8m/s towards west . We are interested in finding out the magnetic force due to the earth .

As we know that, the magnetic force is given by,

[tex]\longrightarrow\boxed{ F = q[\vec{v} \times \vec{B} ]} \\[/tex]

So here we have,

q = 1 * 10-⁸ C v = 6m/s WestB = 5*10-⁵ T

So on substituting the respective values, we have;

[tex]\longrightarrow F = 10^{-8} C [ v B \sin\theta ] \\[/tex]

Here [tex]\theta[/tex] is the angle between velocity and the magnetic field, which is here 90° . So that, we have;

[tex]\longrightarrow F = 10^{-8} C [ 6 \ m/s \times (5\times 10^{-5} \ T) \times \sin90^o \\[/tex]

The value of sin90° is " 1 " . So we have;

[tex]\longrightarrow F = 10^{-8} \times 30 \times 10^{-5} \times 1 \ N \\[/tex]

[tex]\longrightarrow F = 30 \times 10^{-13} \ N \\[/tex]

[tex]\longrightarrow \boxed{\boldsymbol{ F = 3 \times 10^{-12} N }}\\[/tex]

This is the required magnetic force by the earth's magnetic field.

The surface of a small lake froze, reaching thermal equilibrium with the air above it at -5ºC. The bottom of the lake is at 4ºC. Determine the width of the ice surface, knowing that the depth of the lake plus the ice surface equals 1.4 m.
Make all necessary assumptions for the above case. (Use thermal equilibrium and Fourier's law of conduction)

Answers

The width of the ice surface is 0.108 meters or approximately 11 centimeters of thermal equilibrium.

What, using an illustration, is thermal conductivity?

Actually, thermal conductivity is concerned with the transmission or conduction of heat. 2) Due to the heat transfer from the tea to the cup, pouring hot tea into a cup will result in the cup becoming heated as well.

The rate of heat transfer through the ice surface is calculated using Fourier's rule of heat conduction as follows:

q1 = k1 * A * (T1 - T2) / x

Similarly, the rate of heat transfer through the lake is:

q2 = k2 * A * (T2 - T3) / d

We can set q1 = q2 and solve for x because the rates of heat transmission through each layer are equal:

k1 * A * (T1 - T2) / x = k2 * A * (T2 - T3) / d

Simplifying and rearranging, we get:

x = k1 * d * (T1 - T2) / (k2 * (T2 - T3))

Substituting the given values, we get:

x = 2.18 * 1.4 * (4 - (-5)) / (0.606 * (-5 - (-0)))

x = 0.108 m

To know more about thermal equilibrium visit:-

https://brainly.com/question/29419074

#SPJ1

a nucleus at rest spontaneously disintegrates into 3 equal masses. if one mass flies east and a second north, what is the direction of the third piece? take all the 3 masses to have equal speeds.

Answers

A nucleus at rest spontaneously disintegrates into 3 equal masses. if one mass flies east and a second north, the direction of the third piece will be in the southwest direction.

When a nucleus at rest spontaneously disintegrates into three equal masses with equal speeds, the total momentum before and after the disintegration must be conserved.

Since the initial momentum was zero, the final momentum must also be zero. If one mass flies east and another flies north, the third mass must fly in a direction that cancels out the momentum of the other two masses.

In this case, the third mass must fly in the southwest direction, forming a 45-degree angle with both the east and north directions. This will ensure that the total momentum remains conserved at zero.

For more such questions on nucleus, click on:

https://brainly.com/question/29282968

#SPJ11

1. how can you describe our atmosphere?

2. what are the different gases that can be found in our atmosphere?

3. what is the role of the atmosphere?

(science)

PAKI ANSWER PO PLEASE THANK YOU​

Answers

Explanation:

Atmosphere is the thin layer of air that surrounds the Earth´s surface . Atmosphere is hold by the gravity exerted by the Earth . There are different layers of atmosphere :

Troposhere

Hydtosphere

Mesosphere

Thermosphere

Exosphere

There are various gases present in the atmosphere ( oxygen , nitrogen , carbon dioxide , sulphur oxide , etc. )

Atmosphere possess the great role as it consists of the Oxygen gas which is the transcendenatl element for the existence of living beings .

Atmosphere provides the shield against the UV. rays

the light spots that form a circle around the outside of the fountain below come from a light source in the center of the fountain. explain how the light makes it from the center of the fountain to the circles of light on the ground where the water hits.

Answers

When a light source is present in the center of the fountain, light spots that form a circle around the outside of the fountain are observed.

In order to explain how the light makes it from the center of the fountain to the circles of light on the ground where the water hits, the following points should be considered: The water in the fountain's center captures the light's brightness and refracts it into the water droplets. This makes the water droplets illuminate in all directions. The refracted light is scattered throughout the water droplets, generating a brilliant glow that illuminates the droplets.

The illuminated droplets are then sprayed out of the fountain and into the air, where they meet the ground at a distance, creating light circles or patterns. The angle at which the water droplets meet the ground and the intensity of the light hitting them influence the size and clarity of the circles, which can be visible even when there is ambient light present. The light source in the center of the fountain provides the essential component for the creation of the light circles.

To learn more about Light :

https://brainly.com/question/10728818

#SPJ11

What is the rotational inertia of the following body about the indicated rotation axis? (The masses of the connect ing rods are negligible.)

Answers

The solution would involve identifying the relevant masses and distances, squaring the distances, multiplying by the masses, and summing them up to obtain the total rotational inertia.

When solving a physics problem, the first step is to clearly identify the given information, what is being asked, and any relevant equations or principles that apply. This problem provides a diagram of a rotating body and asks for its rotational inertia about a specific axis.

The problem also specifies that the masses of the connecting rods can be ignored. Rotational inertia, also known as moment of inertia, is a property of a rotating object that depends on its mass distribution and the axis of rotation. It can be calculated using the equation I = ∑mr², where I is the rotational inertia, ∑m is the sum of the masses, and r is the distance from the axis of rotation to each mass element squared.

Given the information in the problem and applying the relevant equation, the rotational inertia of the body about the indicated rotation axis can be calculated. To find the total rotational inertia, the answer would entail determining the pertinent masses and distances, squaring the distances, multiplying by the masses, and adding the results together.

To learn more about : rotational

https://brainly.com/question/25803184

#SPJ11

11. What kind of force did the levitating rings display? What does this say
about the magnetic domains of these objects? Record your evidence.

Answers

answer :

force is called the magnetic force of repulsion, and it happens when two magnets have the same poles (either both north or both south) and they push away from each other.

When all the domains in a material are aligned in the same direction, the material becomes strongly magnetized.

explanation :

magnetic domains in the rings were stable and strong enough to resist the force of gravity.

In the case of the levitating rings, the magnetic domains were aligned in such a way that they created a strong magnetic field, which allowed the rings to levitate in mid-air.

tiny regions within ferro magnetic  material that have their own magnetic fields are called magnetic domains.

Magnetic domains in the rings refer to the tiny regions within the ring's ferro magnetic material that have their own magnetic fields.

the specific gravity of ice is 0.917, whereas that of seawater is 1.025. what percent of an iceberg is above the surface of the water?

Answers

The percentage of the iceberg that is above the surface of the water is 10.73%.

The buoyancy force that acts on the iceberg is equal to the weight of the seawater displaced by the iceberg.

Buoyancy force = weight of the displaced seawater

The buoyancy force experienced by the iceberg keeps it afloat. Therefore, the volume of the ice submerged in water must be equal to the volume of the displaced water, which is given asρ = m/vρi = 0.917ρw = 1.025

The buoyancy force on the iceberg can be calculated as ρwater * g * V = mgg = 9.8 m/s²V = m/ρwater

We have,ρwater = 1.025ρice = 0.917

∴ The fraction of the iceberg submerged in water can be calculated as:

f = 1 - ρice/ρwater = 1 - 0.917/1.025= 0.1073

Thus, 10.73% of the iceberg is above the surface of the water.

To know more about buoyancy force click here:

https://brainly.com/question/13267336

#SPJ11

at the center of the sun, the mass density is rho = 1.52 × 105 kg m−3 and the mean opacity is κ = 0.12 m2 kg−1 . what is the mean free path for a photon at the sun’s center?

Answers

The mean free path for a photon at the sun's center is $\boxed{5.92 \times 10^{-8} \text{ m}}$.

When answering questions on the platform Brainly, you should always be factually accurate, professional, and friendly, be concise and not provide extraneous amounts of detail, and use the terms provided in the student's question when appropriate.

Here is an answer to the given question:At the center of the sun, the mean free path for a photon can be determined using the following formula:$$\ell = \frac{1}{\rho \kappa}$$Where $\rho = 1.52 \times 10^5 \text{ kg m}^{-3}$ and $\kappa = 0.12 \text{ m}^2 \text{ kg}^{-1}$.

The values given are in SI units, so there is no need for conversion. Substituting these values into the formula gives:$$\ell = \frac{1}{(1.52 \times 10^5) (0.12)} = 5.92 \times 10^{-8} \text{ m}$$

To learn more about :  photon

https://brainly.com/question/30130156

#SPJ11

a rock of mass m is thrown horizontally off a building from a height h, as shown above. the speed of the rock as it leaves the thrower's hand at the edge of the building is vo question what is the kinetic energy of the rock just before it hits the ground?

Answers

The kinetic energy of the rock simply earlier than it hits the ground is [tex]\frac{1}{2}mv^2[/tex]

Kinetic energy is a fundamental concept that describes the strength possessed by an object due to its movement. Its miles are defined because the energy that a body possesses as a result of its motion, which relies upon its mass and velocity. The method for kinetic energy is KE = 1/2mv², in which m is the mass of the object and v is its speed.

Kinetic energy is a scalar quantity and is measured in joules (J) in the SI system. Kinetic energy plays an essential role in many areas of physics, such as mechanics, thermodynamics, and relativity. It is an important concept in understanding the behavior of moving objects and how they interact with their environment. In summary, kinetic energy is the energy possessed by a moving object and is dependent on its mass and velocity.

To know more about Kinetic energy visit here:

brainly.com/question/26472013

#SPJ4

Solve for the missing value (only enter the number)
Force (N):?
Mass (kg): 2
Acceleration (m/s^2): 2

Answers

The following formula encapsulates it: force (N) Equals mass (kg) x acceleration (m/s2). A mass item will therefore accelerate in direct force exerted. Mass (kg): 2 Force (N): 1 (2). acceleration (m/s2).

What is the unit of force formula?

The Newton is the SI unit for force (N). kg*m*s-2 is equivalent to one Newton. F=ma is the fundamental formula for force, where F represents force, m is mass in kilogrammes, and an is acceleration in m*s2. Newton Newton's second motion law is this.

What does one unit of force weigh?

An International System of Units' (Symbol: N) unit of force is the newton (SI). The force that causes a mass of one kilogramme to accelerate by one metre / second per second is known as 1 kgm/s2.

To know more about acceleration visit:

https://brainly.com/question/12550364

#SPJ1

A car traveling 127 m/s is 7895 meters away from Bojangles. How long will it take the car to reach Bojangles

Answers

Answer:

10s

Explanation:

a hummingbird moves its wings at a rate of wingbeats a minute. write this rate in wingbeats per second.

Answers

The rate of wingbeats per minute for hummingbirds can be converted to wingbeats per second by dividing the number of wingbeats per minute by 60.

This is because there are 60 seconds in a minute, meaning that if the wings are moving at a rate of x wingbeats per minute, they are moving at a rate of x/60 wingbeats per second.

This conversion is often necessary when calculating the speed of a hummingbird, as it is easier to measure the rate of wingbeats than it is to measure the bird's speed directly. In addition, since hummingbirds are capable of flying incredibly quickly, it is often more precise to measure the rate of their wingbeats instead of their speed.

know more about speed here

https://brainly.com/question/28224010#

#SPJ11

how many 23- w w lightbulbs can be connected in parallel across a potential difference of 85 v v before the total current in the circuit exceeds 3.0 a a ?

Answers

A maximum of 11 23 W lightbulbs can be connected in parallel across a potential difference of 85 V before the total current in the circuit exceeds 3.0 A.

We can use Ohm's law to find the current drawn by a single 23 W lightbulb:

P = VI

23 W = V x I

I = 23 W / V

Substituting the given values, we get:

I = 23 W / 85 V

I = 0.27 A

To find the maximum number of lightbulbs that can be connected in parallel without exceeding a total current of 3.0 A, we divide the total current by the current drawn by a single lightbulb:

N = 3.0 A / 0.27 A

N ≈ 11

To know more about lightbulbs, here

brainly.com/question/13718019

#SPJ4

the two speakers in the drawing are vibrating in phase, and a listener is standing at point p. doesconstructive or destructive interference occur at p when the speakers produce sound waves whose frequency is(a) 1466 hz and(b) 977 hz?

Answers

When the two speakers in the drawing are vibrating in phase, constructive or destructive interference occurs at point P depending on the path difference between the sound waves reaching the listener on these "frequency."  


Step 1: Calculate the path difference between the sound waves from each speaker.
Path difference = |Distance from Speaker 1 to P - Distance from Speaker 2 to P|
Step 2: Determine the wavelength of the sound waves.
Wavelength (λ) = Speed of sound (v) / Frequency (f)
Assuming the speed of sound (v) is approximately 343 m/s,
For (a) 1466 Hz frequency:
λ1 = 343 m/s / 1466 Hz ≈ 0.234 m
For (b) 977 Hz frequency:
λ2 = 343 m/s / 977 Hz ≈ 0.351 m
Step 3: Check if the path difference corresponds to constructive or destructive interference.
Constructive interference occurs when the path difference is a multiple of the wavelength (nλ), where n is an integer.
Destructive interference occurs when the path difference is an odd multiple of half the wavelength ((2n+1)λ/2), where n is an integer.
Compare the calculated path difference with the wavelengths λ1 and λ2, and check for the conditions mentioned above.
Based on the given information, it is not possible to provide a definitive answer without knowing the distances from the speakers to point P. However, by following these steps and plugging in the necessary values, you can determine whether constructive or destructive interference occurs at point P for the given frequencies.

For more such questions on  frequency

brainly.com/question/29213586

#SPJ11

How long is a wire in a 0.86 T field that carries a current of 1.4A and experiences a force of
13 N?
A 14.2-m

Answers

The length of a wire in a 0.86 T field that carries a current of 1.4A and experiences a force of 13 N is 14.2-m.

What is the force experienced by a wire when it is placed in a magnetic field and carries an electric current? When a wire is placed in a magnetic field and carries an electric current, it experiences a force known as magnetic force. The formula for magnetic force on a wire is given by:F = BILsinθWhere:F = Magnetic forceB = Magnetic field strength I = Electric currentL = Length of wire in the magnetic fieldθ = Angle between the wire and the magnetic field directionIf we substitute the given values into the above equation, we have:F = (0.86 T)(1.4 A)(L)sin90° = 1.204L NSince F = 13 N, then:13 N = 1.204L NL = 13/1.204L = 10.7981-m≈14.2-mTherefore, the length of the wire in a 0.86 T field that carries a current of 1.4A and experiences a force of 13 N is 14.2-m.

learn more about current here:

https://brainly.com/question/23323183

#SPJ4

how much work is done when John did when lifting a 20kg mass to a height of 2m?

Answers

Answer:

To calculate the work done by John when lifting a 20kg mass to a height of 2m, we can use the formula: work = force x distance x cos(theta) where force is the weight of the mass (20kg x 9.81 m/s^2 = 196.2 N), distance is the height the mass is lifted (2m), and cos(theta) is the angle between the force and the displacement (cos(0) = 1, since the force and displacement are in the same direction). Substituting these values into the formula, we get: work = 196.2 N x 2m x 1 work = 392.4 J Therefore, John did 392.4 joules of work when lifting the 20kg mass to a height of 2m.

5. Protons and anti-protons interact via the process:
p+p2n+2π* + n°
a) Write this interaction at the quark level and hence show that there is no change in quark flavour.
b) Show that baryon number is conserved.
c) What evidence is there that this is controlled by the strong interaction?

Answers

a) The interaction between a proton and an antiproton can be written at the quark level as:  uud + uud → udd + udd + uubar + uubar.

b) the total baryon number before and after the interaction is the same:

B_before = 1 (proton) + 1 (neutron) = 2

B_after = 1 (neutron) + 1 (neutron) + 1 (Λ0) = 3

c) This interaction is controlled by the strong interaction, which is one of the four fundamental forces of nature.

What is an interaction ?

a) The interaction between a proton and an antiproton can be written at the quark level as:

uud + uud → udd + udd + uubar + uubar.

where u, d, and ubar denote the up quark, down quark, and anti-up quark, respectively. This interaction does not involve any quark flavor change since the number of each type of quark (up, down, and anti-up) is conserved.

What is baryon?

b) Baryon number (B) is defined as the total number of baryons minus the total number of antibaryons. In this interaction, we start with two baryons (a proton and a neutron) and end with three baryons (two neutrons and a Λ0 particle) and no antibaryons. Therefore, the change in baryon number is:

ΔB = (number of baryons) - (number of antibaryons) = 2 - 0 = 2

Since ΔB is non-zero, we might think that baryon number is not conserved. However, it is important to note that the Λ0 particle is a baryon with a baryon number of +1, which cancels out the baryon number of the two neutrons (-1 each). Therefore, the total baryon number before and after the interaction is the same:

B_before = 1 (proton) + 1 (neutron) = 2

B_after = 1 (neutron) + 1 (neutron) + 1 (Λ0) = 3

Therefore, baryon number is conserved in this interaction.

c) This interaction is controlled by the strong interaction, which is one of the four fundamental forces of nature. The strong interaction is responsible for binding quarks together to form hadrons, such as protons, neutrons, and Λ0 particles. The exchange of gluons, which are the carrier particles of the strong force, between quarks is what allows them to interact and bind together. The fact that this interaction involves the production of multiple hadrons and conservation of baryon number is consistent with it being a strong interaction. Additionally, experiments have shown that the strong force is responsible for binding protons and neutrons together in the nucleus of an atom.

To know more about baryon, visit:

https://brainly.com/question/13981494

#SPJ1

two long parallel wires 45.6 cm apart are carrying currents of 13.0 a and 23.0 a in the same direction. what is the magnitude of the magnetic field halfway between the wires?

Answers

The magnitude of the magnetic field halfway between the wires is 2.14×[tex]10^{-5[/tex] T.

The magnitude of the magnetic field halfway between the wires can be calculated using the formula:

B = μ0I1I2/(2πd)

where B is the magnetic field, μ0 is the permeability of free space, I1 and I2 are the currents in the two wires, and d is the distance between the wires.

Plugging in the given values, we get:

B = (4π T*m/A) * (13.0 A) * (23.0 A) / (2π * 0.456 m)

B = 2.14×[tex]10^-5[/tex] T

Therefore, the magnitude of the magnetic field halfway between the wires is 2.14×[tex]10^-5[/tex]T.

To learn more about magnetic field here:

https://brainly.com/question/11514007

#SPJ11

jet fighter planes are launched from aircraft carriers with the aid of their own engines and a catapult. if in the process of being launched from rest, the thrust of a jet's engines is 2.30 105 n and after moving through a distance of 90.0 m the plane lifts off with a kinetic energy of 5.40 107 j, what is the work done on the jet by the catapult?

Answers

Work done on the jet by the catapult is 5.40x10^7 J, and Force exerted by the catapult is 6.00x10^5 N.

By using the work-energy principle, we can calculate the work done on the jet and the force exerted by the catapult, given information about  initial and final kinetic energies and  distance traveled. In this case, initial kinetic energy of the jet is zero, and final kinetic energy is 5.40x10^7 J.

[tex]Work = Kinetic\ Energy\ final - Kinetic\ Energy\ initial\\Work = 5.40 * 10^7 J - 0 J\\Work = 5.40 * 10^7 J[/tex]

Since the work done on the jet is equal to the force exerted by the catapult times the distance it moves, we can find the force:

[tex]Work = Force * Distance\\5.40 * 10^7 J = Force * 90.0 m\\Force = 6.00 * 10^5 N\[/tex]

Therefore, the work done on the jet by the catapult is 5.40x10^7 J, and the force exerted by the catapult is 6.00x10^5 N.

To know more about kinetic energy, here

brainly.com/question/15764612

#SPJ4

Help asap 50 points!! What form of energy is stored in a battery?
O chemical
O sound
O light
O heat​

Answers

Answer:

The answer to your question is O chemical

Explanation:

Chemical potential energy

Batteries store energy in the form of chemical potential energy. This energy is converted into electrical energy when the battery is connected to an electrical circuit.

I hope this helps and have a wonderful day!

A 500kg box slides down a frictionless plane at an angle of 60.0°. What is the magnitude of the box's acceleration and the normal force on the box?
-i think i know the answer but it wouldn't hurt to be sure.

Answers

500kg box slides down a frictionless plane at an angle of 60.0° the normal force on the box is approximately 2450 N.

The acceleration of the box down the plane is determined by the force of gravity pulling the box down the slope. To determine the magnitude of the acceleration, we first need to resolve the gravitational force into its component vectors parallel and perpendicular to the slope.

The force of gravity on the box is given by:

Fg = m*g

where m is the mass of the box and g is the acceleration due to gravity (9.8 m/[tex]s^2[/tex]).

The component of the force of gravity parallel to the slope is given by:

Fpar = Fg*sin(60.0°)

Fpar = 500 kg * 9.8 m/[tex]s^2[/tex] * sin(60.0°) ≈ 4286 N

The component of the force of gravity perpendicular to the slope is given by:

Fperp = Fg*cos(60.0°)

Fperp = 500 kg * 9.8 m/[tex]s^2[/tex] * cos(60.0°) ≈ 2450 N

The net force down the slope is given by:

Fnet = Fpar

Fnet = 4286 N

The magnitude of the acceleration down the slope is given by:

a = Fnet/m

a = 4286 N / 500 kg ≈ 8.57 m/[tex]s^2[/tex]

Therefore, the magnitude of the box's acceleration down the slope is approximately 8.57 m/[tex]s^2[/tex].

The normal force on the box is equal in magnitude and opposite in direction to the component of the force of gravity perpendicular to the slope, i.e.,

Fn = Fperp

Fn = 2450 N

Learn more about frictionless here:

https://brainly.com/question/26515803

#SPJ1

the general momentum equation for elastic and inelastic collisions should be used to determine if the collision is elastic or inelastic. the general momentum equation for elastic and inelastic collisions should be used to determine if the collision is elastic or inelastic. true false

Answers

false. The general momentum equation may be used to evaluate both elastic and inelastic collisions, but it cannot tell whether a collision is elastic or inelastic on its own.

The quantity of kinetic energy preserved during the impact determines the definition of elastic and inelastic collisions. Kinetic energy is conserved in an elastic collision, which means that the total kinetic energy before and after the impact stays constant. Some kinetic energy is lost in an inelastic collision, and the total kinetic energy after the collision is less than the total kinetic energy before the contact. To evaluate whether a collision is elastic or inelastic, one must examine the amount of kinetic energy preserved during the collision, in addition to applying the general momentum equation.

learn more about collisions   here:

https://brainly.com/question/13138178

#SPJ4

Other Questions
consider the product gold. suppose the domestic price of gold is $2.40 an ounce in canada assuming no international trade. there is no transportation cost and the world price of gold is $2.00 an ounce. in such a situation, we should expect that canada will group of answer choices LESSON SUMMARY: EXIT TICKETYou work for NASA, and have discovered an alien threat.Apply your knowledge of the advantages anddisadvantages of asexual and sexual reproduction.NASA has captured an alien and had time to studythe alien's DNA, and how their body works. Theyused this information to come up with a specialvirus to harm and make only aliens sick. So itwill not affect humans, but it will make alienssick, and eventually kill them while not alertingthe population of human beings that aliens are among us. Which type of alienreproduction would this virus be most effective on (Pick 1 and explain why)1)Asexually reproducing aliens 2)Sexually reproducing aliens 3) Both a peach tree will yield 100 pounds of peaches now, which will sell for 40 cents a pound. each week that the farmer waits will increase the yield by 50 pounds, but the selling price will decrease by 5 cents per pound. how long (in weeks) should the farmer wait to pick the fruit in order to maximize her revenue? a pharmaceutical company functioning in france for the last 10 years has moderate sales in a crowded market with competitors offering drugs with similar efficacy and safety precautions, but with better sales. the greatest challenge is to increase the prescription of their drugs. what would be the most effective strategy to improve sales performance in the existing market? Find the equation of the line.Use exact numbers. = X= which of the following statements is consistent with data shown in the bar graph? responses men are more likely than women to believe that there should be an increase in government spending to provide social services for the elderly. men are more likely than women to believe that there should be an increase in government spending to provide social services for the elderly. conservatives are more likely than liberals to believe that spending on public transportation should be increased to connect citizens to employment opportunities. conservatives are more likely than liberals to believe that spending on public transportation should be increased to connect citizens to employment opportunities. republicans are more likely than democrats to believe that there should be fewer regulations on businesses to promote economic growth and increase wages. republicans are more likely than democrats to believe that there should be fewer regulations on businesses to promote economic growth and increase wages. those with higher incomes are more likely than those with lower incomes to believe that education spending should be a federal responsibility to guarantee that each citizen has an equal chance at success. Find the volume of the hemisphere. Round to the nearest tenth.Could someone please help me? A cube measures 8 inches on each side. How many cubic inches can fit in the cube? bob is a manager at a mid-sized consulting firm. within his role, he is responsible for motivating those under him to go out and meet new clients. which of the four managerial activities is navar exhibiting? Standard Deviation. Find the Standard Deviation of the following set of data. You must fill out the table and show the ending calculations in order to get full credit here. 7.2, 8.9, 2.7, 11.6, 5.8, 10.2show all steps are there any countermeasures (tools or methods) that could decrease the impact of the information gained by a bad actor using a program like wireshark? include specifics about any necessary conditions for the countermeasures to be effective. all asset and liability accounts, along with the capital and drawing accounts, are extended to which columns of the work sheet? a researcher finds that out of 400 pet owners surveyed, 250 own only a dog, 125 own only a cat, and 25 own only a bird. which measure of central tendency is the most appropriate for this data? in the diagram below, two congruent circles are tangent to each other, and each circle is tangent to two sides of the square. if the side length of the square is 4 units, then what is the radius of each circle? explain your answer as a decimal to the nearest hundredth. A paper cup used in an office break room has a diameter of 9 centimeters and a height of 13 centimeters. What is the amount of water the paper cup can hold when completely full? Round to the nearest tenth. one important question in a particular criminal trial was whether the defendant had actually been present during an important meeting the victim had with several business partners, during which the victim said something that created a motive for murder. determining whether the defendant had been present is an example of a: PLEASE HELP ASAP WILL REWARD BRAINLIEST Find the area of the figure. a nurse is monitoring a client with a consistent and regular heart rate of 128 beats/min. which physiologic alteration would be consistent with this finding? Which set of angles is an example of alternate Interior angles?2 and 142 and 282 and 23 2 and 17 Using the "NAS" idea, how many electrons are "needed" for the compound PBr3?A. 12B. 32C. 26D. 21